Question

Calculate three stocks A, B, C with prices $10, $20, and $50 respectively. a) Calculate a...

Calculate three stocks A, B, C with prices $10, $20, and $50 respectively.

a) Calculate a price-weighted index.

b) The next day, prices change to $12, $18, and $55. What is the new value of the index?

c) What is the return on a portfolio that holds the same number of shares of each stock?

d) Calculate the divisor if stock C splits two for one, using initial prices.

0 0
Add a comment Improve this question Transcribed image text
Answer #1

Answer-

The three stock prices A, B and C have prices $ 10 , $ 20 and $ 50 respectively.

a) The price-weighted index.

price weighted index = sum of the stock prices / divisor
price weighted index = $ 10 + $ 20 + $ 50 / 3
price weighted index = $ 80 / 3 = 26.67

price weighted index = 26.67

b) the prices of A,B and C change to $ 12, $ 18 and $ 55

price weighted index = $ 12 + $ 18 + $ 55 / 3
price weighted index = $ 85 / 3 = 28.33

Therefore the new price weighted index = 28.33

c) The  return on a portfolio that holds the same number of shares of each stock

Suppose there is only one stock of A,B and C
Then return on the portfolio = ( $ 85 - $ 80 ) / $ 80
Then return on the portfolio = $ 5 / $ 80
Then return on the portfolio = 0.0125
Then return on the portfolio = 1.25 %

d) initial prices are A, B and C have prices $ 10 , $ 20 and $ 50 respectively.

Stock C has split by 2:1

Therefore the value of stock C = $ 50 / 2 = $ 25
The index before stock split = ($ 10 + $ 20 + $ 50) / 3 = 26.67
The index after stock split = ($ 10 + $ 20 + $ 25) / 3 = $ 55 / 3 = 18.33

which means that the index decreased although it should not have since stock C's share price did not go down.

We need to keep 26.67 index as it is.

so we have 55 / X = 26.67
Therefore X = 55 / 26.67 = 2.062

X should be lower than 3. It is = 2.062.
Therefore the divisor is 2.062

Add a comment
Know the answer?
Add Answer to:
Calculate three stocks A, B, C with prices $10, $20, and $50 respectively. a) Calculate a...
Your Answer:

Post as a guest

Your Name:

What's your source?

Earn Coins

Coins can be redeemed for fabulous gifts.

Not the answer you're looking for? Ask your own homework help question. Our experts will answer your question WITHIN MINUTES for Free.
Similar Homework Help Questions
  • 2). Consider the following information for three stocks. Time-0 Time= 1 Price per share $90 $50...

    2). Consider the following information for three stocks. Time-0 Time= 1 Price per share $90 $50 $100 Total shares Price er share $108 $90 $55 Total shares outstanding 100 100* 400* Stock outstandingp 100 200 200 "Number of shares outstanding for B and C changed due to reverse and regular stock splits, respectively a) Calculate the value-weighted, equal-weighted, and price weighted index weights of stocks A, B and C at time 0. b) Calculate the holding period return on stocks...

  • the three stocks in the following table. Pe represents price at time t, and Q represents...

    the three stocks in the following table. Pe represents price at time t, and Q represents shares outstanding at time t Stock C splits two for one in the last period. Po A 75 35080 350 80350 B 55 800 50 800 50 800 С 60 900 70 900 351,800 o. Calculate the rate of return on a price weighted index of the three stocks for the first period (t-0 to t-1) (Do not round intermediate calculations. Round your answer...

  • 1. (Total 4 points, one point for cach) Consider the following three stocks in the table...

    1. (Total 4 points, one point for cach) Consider the following three stocks in the table P and Q. denote stock prices and shares outstanding at time I respectively. Stock C splits two for one in the third period. P 0 1 200 75 61 89 20072 150 66 100 90 200 150 100 72 66 45 150 200 a) Calculate the rate of return for Stock A between time and time I assuming that there is no dividend paid...

  • 3. Following is the stock price for three stocks for time 0 and time 1. Time...

    3. Following is the stock price for three stocks for time 0 and time 1. Time 0 Time 1 #Shares 50 100 80 Stock A splits 2-for-1 between time 0 and 1 Stock Stock Price Price # Shares $55 $100 $60 A $30 $115 $40 A 100 B 100 80 C C What is the value of a price weighted index including all three stocks at time 0? (3 points) a) b) What is the new divisor for a price-weighted...

  • Please include work and formulas 2). Consider the following information for three stocks Time -0 Time...

    Please include work and formulas 2). Consider the following information for three stocks Time -0 Time -1 Total shares outstanding 100 200 200 Number of shares outstanding for B and C changed due to reverse and regular stock splits, Price per share S90 S50 $100 Price per share S108 $90 $55 Total shares outstanding 100 100* 400* Stock respectively a) Calculate the value-weighted, equal-weighted, and price weighted index weights of stocks A, B and C at time 0 b) Calculate...

  • A price-weighted index consists of stocks A, B, and C which are priced at $55, $38,...

    A price-weighted index consists of stocks A, B, and C which are priced at $55, $38, and $17 a share, respectively. The current index divisor is 2.5. What will the new index divisor be if stock A undergoes a 5-for-1 stock split?

  • Consider the three stocks in the following table. Pt represents price at time t,

    Consider the three stocks in the following table. Pt represents price at time t, and Qt represents shares outstanding at time t. Stock C splits two-for-one in the last period.a) Calculate the rate of return on a price-weighted index of the three stocks for the first period (from t=0 to t=1). b) What must happen to the divisor for the price-weighted index in year 2? c) Calculate the rate of return of the price-weighted index for the second period (from t =...

  • A value-weighted index consisting of stocks A, B, and C was created yesterday. When the index...

    A value-weighted index consisting of stocks A, B, and C was created yesterday. When the index was created, stocks A,B, and C traded for $80, $45, and $125, respectively. The number of shares outstanding for A,B, and C, was 500, 900, and 600 when the index was formed. Today, stocks A, B, and C trade for $65, $50, and $145, respectively. Find the return on the index from yesterday to today Round intermediate steps and your final answer to four...

  • A market value weighted index has three stocks in it, call them A, B, and C,...

    A market value weighted index has three stocks in it, call them A, B, and C, priced at 54, 60, and 27 per share. Each firm has 339, 376 and 421 thousand shares outstanding, respectively. The value of the index at close of trading day is 834. At this time, the index decides to remove stock C from the index, and in its place to insert stock D. Stock D has a closing price of $85 per share, and 196...

  • 9. Consider the three stocks in the following table. P, represents price at time t, and...

    9. Consider the three stocks in the following table. P, represents price at time t, and Q, represents shares outstanding at time t. Stock C splits two-for-one in the last period. (LO 2-2) Pt 95 45 110 100 200 200 100 200 400 95 90 50 100 100 200 200 45 a. Calculate the rate of return on a price-weighted index of the three stocks for the first period (i 0 to t 1). b. What must happen to the...

ADVERTISEMENT
Free Homework Help App
Download From Google Play
Scan Your Homework
to Get Instant Free Answers
Need Online Homework Help?
Ask a Question
Get Answers For Free
Most questions answered within 3 hours.
ADVERTISEMENT
ADVERTISEMENT
ADVERTISEMENT